Foro de preguntas y respuestas de matemáticas, de cualquier nivel. Cuánto más interesantes, divertidas o intrépidas, mejor.
Aviso: Te invitamos a conocer la página de Facebook de la UCIM

Ganas puntos al hacer preguntas, contestarlas y, sobre todo, si tu respuesta es seleccionada como la mejor.
Registrate como usuario para participar en el foro. También puedes utilizar tu identidad de FB Utiliza el botón azul para ingresar (si usas tu identidad de FB y estás logeado en FB, automáticamente te reconoce).

El irracional tiene una página en FB. El Irracional






+1 voto

Denotemos con $\mathbb{P}$ al conjunto de los números primos. 

Demuestre que para todo número natural $n$ se cumple que $$\prod_{p \in \mathbb{P}, \, p \leq n}p \leq 4^{n-1}.$$

Sugerencias. Haga inducción completa sobre $n$. En el paso de inducción será de ayuda notar que si $k \in \mathbb{N}$ entonces $\prod_{k+1<p\leq 2k+1}p$ divide al coeficiente binomial $\binom{2k+1}{k}$.

(Para la audiencia de las "Degustaciones Matemáticas": a) Una solución al problema vale por un lápiz y/o bolígrafo del evento. b) Intente reconstruir la prueba de la igualdad $\lim_{n \to \infty} \frac{\pi(n)}{n}=0$ que mediante Erdös-Kalmár dimos en la charla. )

por (39,8m puntos) en Básicas
Licencia Creative Commons
Este obra está bajo una Licencia Creative Commons Atribución-NoComercial-CompartirIgual 2.5 México.

powered by UCIM  -  Aviso de privacidad

...